Вы находитесь на странице: 1из 333

Introduction to

transport phenomena
Contents
1. Introduction to transport phenomena
2. Vector and Tensor analysis
3. Viscosity and mechanism of Momentum transport
4. Equations of change for isothermal systems
5. Energy Transport
6. Mass Transport
7. Analogies in heat, mass and momentum transport
Transport Phenomena
What exactly are "transport phenomena"?

Transport phenomena are really just a group of three areas of


study that have certain ideas in common.
These three areas of study are:
• Fluid mechanics (Momentum transfer)
• Heat transfer
• Mass transfer
o They are grouped together due to their:
o Similar mathematical framework,
o Similar molecular mechanism through which the transport
of these quantities may be visualized, and
o Importance to all engineering disciplines.
Transport Processes
Momentum Transport – transfer of momentum which
occurs in moving media (fluid flow, sedimentation, mixing,
filtration, etc.)
Heat Transport – transfer of energy from one region to
another (drying, evaporation, distillation)
Mass Transport – transfer of mass of various chemical
species from one phase to another distinct phase
(distillation, absorption, adsorption, etc.)

 One of the cornerstones of engineering education


Why Study Transport Phenomena?
Why Study Transport Phenomena?
Why Study Transport Phenomena
Chemical
Engineering
Thermodynamics

Transport
Materials
Phenomena
Science
PROCESS
EQUIPMENT
DESIGN

Chemical
Reaction Process
Kinetics Economics
Examples
Consider a two phase continuous
A
stirred tank reactor.
• The reaction is of the form A going Catalyst
to B, and is catalyzed by a solid
catalyst.
• AB
• A is converted to B at the surface
of the solid catalyst B

• Continuous reaction Sufficient A


Rate of transport of A, B and heat
to and from the surface of the B ± Heat
catalyst
•  Mass transport
Examples
• Exothermic reaction, the reaction generates heat.
o The rate at which heat is transferred A
away from the surface should ensure
that no net heat accumulation at the
surface results in a temperature
increase to avoid a hot spot.
• Endothermic reaction, heat is
necessary in order for the reaction to
take place. B
A
o Ensure sufficient rate of transport of
energy to the surface, to avoid
cooling down of the interface so that, B ± Heat

the reaction rate will not decrease,


o  Heat Transport
Examples
• Mixing: in order for the reaction to
A
take place sufficiently fast, there
has to be adequate mixing.
• This mixing is carried out in this
case through the use of an
impeller which is rotating within
the fluid.
• Power input, to compensate B
energy loss due to the fluid A
friction
•  Momentum transport B ± Heat
Examples
• Consider a shell and tube heat
exchanger
• Hot fluid coming in, and it gets
cooled by the coolant,
• Ensure that there is sufficient heat
taken out of the hot fluid so that the
final temperature is the desired one.
• Heat taken out can be used for
other useful purposes.
• Heat transfer depends upon: the
temperature of the hot fluid, the
temperature of the coolant, the fluid
speed, material of the tube.
Examples
• For fluid to flow through the tube, it is necessary to apply a
pressure difference across the ends of the tube
• The longer the tube the larger the pressure drop
• Therefore, during the exchanger design, we have to
consider:
o The heat transfer rates based upon the temperature
differences, fluid properties and material of tube
o The flow rates (Mass transfer)
o The pressure difference and the length of the tube,
(linked with it is also the momentum transfer rate)
TRANSPORT PHENOMENA - Summary
What are Transport Phenomena?
They include three closely related phenomena
o Momentum transport: Fluid Mechanics
o Energy transport: Heat Transfer
o Mass transport: Mass Transfer
• Why studied together?
o Occur simultaneously in many disciplines of engineering
o Basic equations describing them are closely related
o Mathematical tools needed to describe them are similar
o Molecular mechanisms underlying the various transport
phenomena are closely related
Fundamental Quantities
• Flux =Amount transfer per unit area per unit time
• Mass Flux J = Mass transfer per unit area per unit time
• Heat Flux q =Heat transfer per unit area per unit time
• Momentum Flux t = Momentum transfer per unit area per
unit time
= Stress (F/A=mdv/Adt )
• Driving forces
o Mass transfer – Concentration gradient
o Heat transfer – Temperature gradient
o Momentum Velocity gradient
Levels of analysis
Levels of Analysis
• MACROSCOPIC
o Use of macroscopic balances
o Overall assessment of a system

• MICROSCOPIC
o Small region/volume element is selected
o Use of equations of change
o Velocity, temperature, pressure and concentration
profiles are determined

• MOLECULAR
o Molecular structure and intermolecular forces
become significant
o Complex molecules, extreme T and P, chemically
reacting systems
Vectors
SCALARS, VECTORS AND TENSORS

A scalar has magnitude but no direction.


An example is pressure p.
The coordinates x, y and z of Cartesian space are scalars.
A vector has both magnitude and direction
Let î, ĵ, k̂ denote unit vectors in the x, y and z direction. The hat
denotes a magnitude of unity

The position vector x (the arrow denotes a vector that is not a unit
vector) is given as
z
 
x  x î  yĵ  zk̂ x

î ĵ y
x
18
SCALARS, VECTORS AND TENSORS

The velocity vector u is given as

 dx dx dy dz
u  î  ĵ  k̂
dt dt dt dt

The acceleration vector a is given as
 
 du du dv dw d2x d2x d2y d2z
a  î  ĵ  k̂  2  2 î  2 ĵ  2 k̂
dt dt dt dt dt dt dt dt
The units that we will use in class are length L, time T, mass M and
temperature °. The units of a parameter are denoted in brackets. Thus

[x ]  L

[u]  LT1

[a ]  ? LT2

Newton’s second law is a vectorial statement: where F denotes the
force vector and m denotes
  the mass (which is a scalar)
F  ma
19
SCALARS, VECTORS AND TENSORS

The components of the force vector can be written as follows:



F  Fx î  Fy ĵ  Fzk̂

The dimensions of the force vector are the dimension of mass times
the dimension acceleration

[F]  [Fx ]  MLT 2

Pressure p, which is a scalar, has dimensions of force per unit area.


The dimensions of pressure are thus
[p]  MLT 2 /(L2 )  ML1T 2

The acceleration of gravity g is a scalar with the dimensions of (of


course) acceleration:

[g]  LT202
SCALARS, VECTORS AND TENSORS

A scalar can be a function of a vector, a vector of a scalar, etc. For


example, in fluid flows pressure and velocity are both functions of
position and time:
   
p  p(x, t) , u  u(x, t)

A scalar is a zero-order tensor. A vector is a first-order tensor. A


matrix is a second order tensor. For example, consider the stress
tensor t.

 t xx t xy t xz 
 
t   t yx t yy t yz 
t t zy t zz 
 zx

The stress tensor has 9 components. What do they mean? Use the
following mnemonic device: first face,
21 second stress
SCALARS, VECTORS AND TENSORS
Consider the volume element below.
z

Each of the six faces has a direction.


For example, this face
and this face
are normal to the y direction

A force acting on any face can act in the x, y and z directions.


22
SCALARS, VECTORS AND TENSORS
Consider the face below.
z

tyy tyz
y
tyx
x
The face is in the direction y.
• The force per unit face area acting in the x direction on that face is
the stress tyx (first face, second stress).
• The forces per unit face area acting in the y and z directions on that
face are the stresses tyy and tyz.
• Here tyy is a normal stress (acts normal, or perpendicular to the
face) and tyx and tyz are shear stresses (act parallel to the face)
23
SCALARS, VECTORS AND TENSORS
Some conventions are in order
z

tyx
tyy tyz tyy
tyz y
tyx
x

Normal stresses are defined to be positive outward, so the orientation


is reversed on the face located y from the origin
Shear stresses similarly reverse sign on the opposite face face are the
stresses tyy and tyz.
Thus a positive normal stress puts a body in tension, and a negative
normal stress puts the body in compression. Shear stresses always put
the body in shear.` 24
SCALARS, VECTORS AND TENSORS
Another way to write a vector is in Cartesian form:

x  x î  yĵ  zk̂  (x, y, z)
The coordinates x, y and z can also be written as x1, x2, x3. Thus the
vector can be written as

x  (x1, x 2, x3 )
or as

x  (xi ) , i  1..3
or in index notation, simply as

x  xi
where i is understood to be a dummy variable running from 1 to 3.

Thus xi, xj and xp all refer to the same vector (x1, x2 and x3) , as the
index (subscript) always runs from 1 to 3.
25
SCALARS, VECTORS AND TENSORS

Scalar multiplication: let  be a scalar and A = Ai be a vector.

Then A  Ai  (Ai, A2, A3 ) is a vector.
Dot or scalar product of two vectors results in a scalar:
 
A  B  A1B1  A2B2  A3B3  scalar
In index notation, the dot product takes the form
  3 3 3
A  B   Ai Bi   Ak Bk   Ar Br 
i 1 k 1 r 1

Einstein summation convention: if the same index occurs twice, always


sum over that index. So we abbreviate to
 
A  B  AiBi  AkBk  ArBr
There is no free index in the above expressions. Instead the indices are
paired (e.g. two i’s), implying summation. The result of the dot product
is thus a scalar. 26
SCALARS, VECTORS AND TENSORS
Magnitude of a vector: 2  
A  A  A  Ai Ai

A tensor can be constructed by multiplying two vectors (not scalar


product):
 A1B1 A2 B1 A3 B1 
 
Ai B j  ( Ai B j ) , i  1..3, j  1..3   A1B2 A2 B2 A3 B3 
AB A3 B3 
 1 3 A2 B3

Two free indices (i, j) means the result is a second-order tensor


Now consider the expression
Ai A jB j
This is a first-order tensor, or vector because there is only one free
index, i (the j’s are paired, implying summation).
Ai A jBj  ( A1B1  A227B2  A3B2 )( A1, A2, A3 )
That is, scalar times vector = vector.
SCALARS, VECTORS AND TENSORS
Kronecker delta ij

 1 0 0
1 if i  j  
ij    0 1 0
0 if i  j  
 0 0 1 
Since there are two free indices, the result is a second-order tensor, or
matrix. The Kronecker delta corresponds to the identity matrix.

Third-order Levi-Civita tensor.

 1 if i, j,k cycle clockwise: 1,2,3, 2,3,1 or 3,1,2



ijl   1 if i, j,k cycle counterclockwise: 1,3,2, 3,2,2 or 2,1,3
 otherwise
 0
Vectorial cross product:
 
AxB  ijkA jB
28k

One free index, so the result must be a vector.


SCALARS, VECTORS AND TENSORS

Vectorial cross product: Let C be given as
  
C  AxB
Then

 î ĵ k̂  î ĵ k̂
  
C  det A1 A 2 A 3   A1 A 2 A3 
 
 B1 B2 B3  B1 B2 B3

 î ĵ k̂  î ĵ   î ĵ k̂  î ĵ 
     
 A1 A 2 A 3   A1 A 2    A1 A 2 A 3   A1 A2  
     
 B1 B2 B3  B1 B2   B1 B2 B3  B1 B2 

A2B3  A3B2 î  A3B1  A1B3 ĵ  A1B2  A2B1k̂


29
SCALARS, VECTORS AND TENSORS
Vectorial cross product in tensor notation:
Ci  ijkA jBk
Thus for example
=1 = -1 =0
C1  1jkA jBk  123A2B3  132A3B2  111A1B1  a lot of other terms that
all = 0
 A2B3  A3B2
i.e. the same result as the other slide. The same results are also
obtained for C2 and C3.

The nabla vector operator  :
   
  î  ĵ  k̂
x1 x 2 x 3
or in index notation 
x i 30
SCALARS, VECTORS AND TENSORS
The gradient converts a scalar to a vector. For example, where p is
pressure,
 p p p
grad(p)  p  î  ĵ  k̂
x1 x 2 x 3
or in index notation
p
grad(p) 
xi
The single free index i (free in that it is not paired with another i) in the
above expression means that grad(p) is a vector.

The divergence converts a vector into a scalar. For example, where u
is the velocity vector,
 u1 u2 u3 ui uk
div(u)     
x1 x 2 x 3 xi xk
Note that there is no free index (two i’s or two k’s), so the result is a
31
scalar.
SCALARS, VECTORS AND TENSORS

The curl converts a vector to a vector. For example, where u is the
velocity vector,
î ĵ k̂
     
curl(u)  xu  
x1 x 2 x 3
u1 u2 u3
 u3 u2   u1 u3   u2 u1 
   î     ĵ    k̂
 x 2 x 3   x 3 x1   x1 x 2 
or in index notation,
 uk
curl(u)  ijk
x j
One free index i (the j’s and the k’s are paired) means that the result is a
vector
32
SCALARS, VECTORS AND TENSORS

A useful manipulation in tensor notation can be used to change an index


in an expression:

ijuj  ui

This manipulation works because the Kronecker delta ij = 0 except when
i = j, in which case it equals 1.

33
MECHANISMS OF
MOMENTUM TRANSPORT
Topics
• Definitions
• Properties at a point
• Molecular Transport of Momentum
• Convective Momentum Transport
Definitions
• Fluid: as a substance which deforms continuously under
the action of a shear stress.

• Then when a fluid is at rest, there can be no shear stress


Concept of a continuum
• Most engineering work is concerned with the
macroscopic or bulk behavior of a fluid rather than
with the microscopic or molecular behavior.
• In most cases it is convenient to think as a
continuous distribution of matter or a continuum.
• Validity of this concept is to be dependent upon
the type of information desired rather than the
nature of the fluid.
Definitions
• Steady state flow – physical properties of the fluid does
not change.
• Unsteady state flow – the properties change with time in
any point of the system
Definitions
 Uniform flow – when the velocity at any point,
across transversal section in the direction that
motion occurs is constant. In other words when
the velocity vector are parallel and preserved their
intensity
 Non uniform – when the velocity vectors are not
parallel and did not maintain their intensity
constant. Example: The flow fluid through a tube
of constant diameter is uniform, but the flow
through a tube of variable diameter (conic form) is
not uniform.
Properties At a Point
• Density: It is defined as the mass per unit
volume. Under flow conditions, particularly in
gases, the density may vary greatly throughout
the fluid.
• The density, ρ, at a particular point in the fluid is
defined as m
  lim
V  V V
• Where Δm: mass contained in the volume ΔV
• δV is the smallest volume.
Properties At a Point (Cont.)
Stress:
• Consider the force ΔF acting on an element ΔA of the body
shown in the figure.
• The force ΔF is resolved into components normal and
parallel to the surface of the element.
• The force per unit area or stress at a point is defined as
the limit of ΔF/ΔA as ΔA→δA
Force on an element of fluid
ΔF

ΔFn
ΔFs
Fn
lim   ii
A A A

Fs ΔA
lim   ij
A A A

• Where σii is called the normal stress and  ij is


the shear stress (Flux tensor)
Molecular Transport of Momentum
• The buildup to the steady, laminar velocity profile for a fluid
contained between two plates.
• Each of the plates have an area A, separated by a
distance Y.
• In the space let’s consider there is a fluid (gas or liquid).
Build-up to the steady-state
laminar velocity profile

Y t<0 vx(y,t) Small t

vx(y)
t=0 Large t

V
• In this Figure :
o First: The system is at rest, but a time t= 0
o Second: At t= 0 the lower plate is set in motion in the
direction at a constant velocity
o Third: At time proceeds (Small t), the fluid gain
momentum
o Fourth: Ultimately the linear steady-state velocity
profile shown in the figure is established.
 At steady-state motion, a constant force F is
required to maintain the motion of the lower plate.
The force is proportional to the area and to the
velocity and inversely proportional to the distance
between plates.
F V
  (1)
A Y
 Where F/A is  , which is the force in the
direction x per unit area perpendicular to the y
direction.
 It is the force exerted by the fluid of lesser y on the
fluid of greater y, therefore V/Y can be replaced by
–dvx/dy.
• The equation can also be written as
dvx (2)
 yx 
dy
Newton’s Law of viscosity

 Where:
 μ: viscosity of fluid, (Pa.s)
yx : Flux tensor (shear stress) in the positive y
direction , [(N/m2)= Pa]
 vx: velocity in x direction, (m/s)
 y: distance, (m)
Generalization of Newton’s Law of viscosity
• The equation (2) was defined only in terms of a simple
steady state shearing flowing in which vx as a function of y
alone, and vy and vz are zero.
• This situation is not really so common. Usually the system
is composed by a flow in which the three velocity
components may depend on all three coordinates and
possibly on time.
Consider
• A general flow pattern in which the velocity shows in
various directions depends on the time.
• The velocity components are given by

vx  vx ( x, y, z, t );
vy  v y ( x, y, z, t ),
vz  v2 ( x, y, z, t )
• There will be then, 9 stress components (where i and j
may be taken on the designations (x, y, z), instead of the
component that appears in equation (2)
 yx  ij
Pressure and viscous forces acting on
planes

z
x, y, z

x
• The volume element can be cut to each of three
coordinates in turn.
• Find the forces that have been exerted on that surface by
the fluid that was removed.
• There will be two forces that contribute
o Associated with the pressure
o Associated with the viscous forces
• Pressure force
o Always perpendicular to the exposed surface
• For example in x direction the force will be a vector pδx that is the
pressure (a scalar) multiplied by the unit vector δx in the x
direction. Similarly for the other sections.
o Pressure forces will be exerted when the fluid is
stationary as well as when it is in motion
Viscous forces
• They exist only when there are velocity gradients
within the fluid.
• They are neither perpendicular nor parallel
 xto the
surface element, rather at some angle to the
surface.
• Those forces represent in the Figure are vectors
with scalar components.
o For example,  x has components  xx , xy , xz
• Pressure and Viscous
Forces acting on y
planes

x x
pδy

pδx

z pδz
Two Ways to Interpret the definition

 ij  p ij   ij
• force in the j direction on a unit area to the i
direction, where it is understood that the fluid in
the region of lesser xi is exerting the force on the
fluid of a greater xi.
• flux of j-momentum in the positive (+) i direction,
It is understood that is, from the region of lesser
xi to that of greater xi.
Convective Momentum Transport
• The momentum can be transported by bulk flow of
the fluid. This process is called convective
transport. Use the same figure on three planes
• The fluid vector at the centre is v
• The volume rate of flow across the area in the first
figure is vx. The fluid carries with it momentum ρv
per unit volume.
• Hence the momentum flux across this area is then
vxρv (vectors)
• Convective momentum
fluxes through planes
of unit area
perpendicular to the
coordinates direction x
pvyv

pvxv

pvzv
Shell momentum
balance and Velocity
distribution
Laminar Flow
Shell Momentum Balance
• Volume element
• Consist in the establishment of a region in the
space, within a specific system, at which the
fundamental physical laws are applied to obtain
equations to describe the phenomena that occur in
the system.
• It is a region characterized with a constant volume.
• The limit surfaces that limit this volume are called
control surfaces.
Volume control

L Solid surface

Volume control Flow


B

Δx

Flow system of volume control


Momentum Balance to thin shell

rate  rate  rate  rate   force 


of  of  of    of 
       of   
momentum  momentum   momentum  momentum   gravity 
     0
in by  out by  in by  in  acting 
convective  convective  molecular  molecular  on 
         
trasnport  trasnport  trasnport  trasnport   system 
• This a restricted statement of the law of conservation of
momentum
• To write the momentum balance we need the expressions
for the convective and molecular momentum fluxes
• In summary this general equation can be
expressed as

 Sum 
 
 flow   flow  of 
     forces 
density  density   
   
of   of   that are   0
momentum  momentum  acting 
     
in  out  on 
 system 
 
• Flow density
o Momentum at entry or exit the system by molecular
transport or by global fluid motion.
• Sum of forces
o Gravity force
o Pressure
Procedure to set and solve viscous flow
problems
 Identify the no vanishing velocity component and
the spatial variable on which it depend.
 Write a momentum balance over the thin shell
perpendicular to the relevant spatial variable.
 Let the thickness of the shell approach zero and
make use of the definition of the first derivative to
obtain the corresponding differential equation for
the momentum flux.
Procedure to set and solve viscous flow
problems (Cont.)
• Integrate this equation to get the momentum flux
distribution.
• Insert Newton’s Law of viscosity and obtain a
differential equation for the velocity.
• Integrate this equation to get the velocity
distribution.
• Use the velocity distribution to get other quantities,
such as the maximum velocity, average velocity,
or force on solid surface.
Boundary conditions
• Considerations
o There is not material passing through the interface
(there is not adsorption, absorption, dissolution,
evaporation, melting or chemical reaction at the surface
between the two phases.
Boundary conditions
• At solid-fluid interfaces
• The fluid velocity is equal the velocity with which the solid
surface is moving.
• This statement is applied to both the tangential and normal
component of the velocity vector.
• The base of this statement can be found in the roughness
of the surface and the adhesion forces between solid and
fluid.
Boundary Conditions
• At liquid-liquid interfacial plane
• The velocity and flux tensor have the same value in both
faces.
• The flow density (stress-tensor) and the velocity are
continuous through the interface
Boundary Conditions
• At liquid-gas interfacial plane of constant x, the
stress-tensor components  xy and  xz are taken to
be zero.
• The gas side velocity gradient is not too large
• The momentum is practically nor being transfer,
therefore for calculation purposes can assumed
that
 dv 
  0 or    0
 dy 
Flow of a falling film


L

Solid Surface Liquid film

Zone when the


velocity profile is
disrupted
Volume Element for a falling film
Momentum In

L
Width = B x
y Volume
element

Momentum
Out
Δx
Gravity
force
Considerations
• Isothermal, laminar and stationary flow of a falling film
down an inclined flat plane of length L and width B.
• The plane is α angle with respect of the gravity force.
• Newtonian fluid
• The viscosity and density are constant.
Considerations (Cont.)
• The length (L ) is sufficiently small or taken very
far from both extreme, therefore the velocity
distribution is not affected.
• The film has a small thickness δ in x-direction and
B in z-direction.
• The volume element has a width B and finite
thickness Δx.
• The force that produce the fluid motion is a
component of the gravity force – gy= ρcosα
Considerations (Cont.)
• The direction of the flow is taken in the positive
direction of the coordinates axis.
• The flow density  xy at the entry and exit of the
volume element is evaluated at the position with
respect to the location of coordinates axis
(molecular transport).
Considerations
• The flow density at the entry and exit of the volume due to
the bulk of fluid is evaluated at the position with respect to
the location of coordinates axis (convective transport). It is
represented by ρvyvy. This magnitude is also evaluated for
the inlet and outlet points for y = 0 and y = L.
• Applying the general equation of momentum balance and
multiplying each term by the magnitude that transform this
balance of momentum in force balance

 
 BL   xy x  xy 
 
 1 2 x x 

 
( Bx)   v y2   v 2 
 y 0 y

 3 4 yL 

 
 BLx    g cos    0
 5 
• Terms 3 and 4 are the equal (ρvyvy) in y =0 and y
=L. Can be eliminated.
• Therefore the equation is
   
 BL   xy x  xy    BLx    g cos    0
 
 1 2 x x   5 
multiplying by (1/ BLx) and taken

 xy x  xy
x x
  g cos 
1 2
lim
x 0 x
• The left term of the equation by definition is the
first derivate of  xy with respect to x, therefore,
d xy
  g cos 
dx
• Integrating  xy  (  g cos  ) x  C1

• Applying Boundary Conditions (BC)


o Liquid – Gas; x =δ and  xy =0
 xy equation we obtain
• Substituting in the previous
the Flux tensor distribution equation

 xy  (  g cos  )( x   )
Velocity Distribution
• Based on the assumption of Newtonian fluid,
then,
dv
xy  y ((ggcos
dv y
cos)()(xx))
xy dx
dx
dvyy  ((ggcos
dv
cos))(  x)
dx   (  x)
dx 
int
integrating
• Integrating, egrating
((ggcos 
 ) x 2
2
vvy  cos )((xx ))CC2
x
y  22 2
• Applying BC:2; x = 0 and vy = 0, then C2=0

(  g cos  ) 2
x
vy  ( x  )
 2
• Using this expression we can determine the
maximum velocity, average velocity, mass flow
rate, film thickness and force of the fluid in the
solid surface.
• Maximum Velocity (x =δ and (dvy/dx) = 0
(  g cos  )
2
v ymax 
2
• Average velocity
• Mass flow rate
B
w    v y dxdz  B v y
0 0

• Force per unit area

L B
Fy    ( xy x  ) dxdy
0 0
Flow Through a Circular Tube

Liquid is flowing across a pipe


of length L and radius R.

Assumptions:
1. Steady-state flow
2. Incompressible fluid
3. Only Vx component is significant
4. At the solid-liquid interface, no-slip condition
Recall: Cylindrical Coordinates
Flow Through a Circular Tube

 rate of momentum   rate of momentum 


     force of gravity 
 in by molecular 
  out by molecular    acting on system   0
 transport   transport   
   

pressure : PA z 0  PA z L
net momentum flux :  rz A1 r   rz A2 r r

Adding all terms together:


P  2 r r  z 0  P  2 r r  z  L   rz  2 rL  r   rz  2 rL  r r  0
Flow Through a Circular Tube

P  2 r r  z 0  P  2 r r  z L   rz  2 rL  r   rz  2 rL  r r  0
Dividing by 2 Lr :
 P z  0  P z  L   rz r r   rz r r  r
  r  0
 L  r

Let x  0 :
 P0  PL  d
  r   rz r   0
 L  dr
Flow Through a Circular Tube

 P0  PL  d
  r   rz r   0
 L  dr

Solving: BOUNDARY CONDITION!


d  P0  PL  At the center of the pipe, the flux is
 rz  
 r  r zero (the velocity profile attains a
dr  L  maximum value at the center).
 P0  PL  2
 rz r    r  C1 C1
 2L  00 C1 must be

 P0  PL  C1 r zero! 
 rz   r 
 2L  r
Flow Through a Circular Tube
 P0  PL 
 rz   r
 2L 
From the definition of flux: BOUNDARY CONDITION!
dv At r = R, vz = 0.
 rz    z
dr
P P 
0    0 L  R2  C2
 4 L 
Plugging in: P P 
C2   0 L  R2
dv z  P0  PL   4 L 
  r
dr  2L 
 P0  PL  2  P0  PL  2
 P0  PL  2 vz    r   R
vz     r  C2  4 L   4 L 
 4 L 
Flow Through a Circular Tube

 P0  PL  2 2
vz    R  r 
 4 L 

Compute for the following:

Average Velocity:
2 R

zv dA   v rdrd z
v z  v z , ave  0

0

 dA   rdrd
2 R

0 0
Hagen-Poiseuille Equation

 P0  PL  2
v ave  D
 32L 

Describes the pressure drop and


flow of fluid (in the laminar regime)
across a conduit with length L and
diameter D
What if…?
The tube is oriented
vertically.

What will be the velocity


profile of a fluid whose
direction of flow is in the
+z-direction
(downwards)?
Non-Newtonian Flows
Newtonian Fluids

water
air

ethyl
alcohol
Non-Newtonian Fluids

blood

ketchup
toothpaste
Non-Newtonian Fluids

grease

polymer melt
cake batter
Non-Newtonian Fluids

molten metal paint

whipped cream
Non-Newtonian Fluids
• Foods
o Emulsions (mayonnaise, ice cream)
o Foams (ice cream, whipped cream)
o Suspensions (mustard, chocolate)
o Gels (cheese)
• Biofluids • Electronic and Optical Materials
o Suspension (blood) – Liquid Crystals (monitor
o Gel (mucin) displays)
o Solutions (spittle) – Melts (soldering paste)
• Personal Care Products • Pharmaceuticals
o Suspensions (nail polish, face scrubs) – Gels (creams, particle
precursors)
o Solutions/Gels (shampoos,
conditioners) – Emulsions (creams)
o Foams (shaving cream)
– Aerosols (nasal sprays)
• Polymers
Non-Newtonian Fluids

Why are these fluids non-Newtonian?

Non-Newtonian behavior is frequently associated


with complex internal structure:
• The fluid may have large complex molecules (like
a polymer), or
• The fluid may be a heterogeneous solution (like a
suspension)...
Non-Newtonian Fluids

Why are these fluids non-Newtonian?

Fluid systems may be non-ideal in two ways:


1. The viscosity may depend on shear rate
2. The viscosity may depend on time
Some (many) may have both
Non-Newtonian Fluids
• Any fluid that does not obey the Newtonian relationship between the
shear stress and shear rate is called non-Newtonian.


  f 
τ
2
0 – Newtonian fluid
1 – Bingham plastics
2 – pseudoplastic fluids
0 3 – dilatant fluids
slope η
3

• When the viscosity decreases with increasing shear rate, we call the
fluid shear-thinning.
• In the opposite case where the viscosity increases as the fluid is
subjected to a higher shear rate, the fluid is called shear-thickening.
• Shear-thinning behavior is more common than shear-thickening.
• Shear-thinning fluids also are called pseudoplastic fluids.
• We describe the relationship between the shear stress
and shear rate as follows.
  
• Here, η is called the “apparent viscosity” of the fluid, and is
a function of the shear rate.
• Many shear-thinning fluids will exhibit Newtonian behavior
at extreme shear rates, both low and high.
• For such fluids, when ln(apparent viscosity) is plotted
against ln(shear rate), we see a curve like this.
Non-Newtonian Fluids

• The regions where the apparent viscosity is approximately constant


are known as Newtonian regions. The behavior between these regions
can usually be approximated by a straight line on these axes. It is
known as the power-law region.
• In this region, we can approximate the behavior by

• Or

• Instead of b we commonly use for the exponent (n-1) and


write a result for the apparent viscosity as follows.

• Then, we get the power-law model.


• where n is called the power-law index. Note that n=1
corresponds to Newtonian behavior. Typically, for shear
thinning fluids, n lies between 1/3 and1/2.
Fluid flow in a plane narrow slit
(Newtonian + Non-Newtonian)
• Consider a fluid (of density ρ) in incompressible, laminar flow in a
plane narrow slit of length L and width W formed by two flat parallel
walls that are a distance 2B apart. End effects may be neglected
because B << W << L. The fluid flows under the influence of both a
pressure difference Δp and gravity.

a) Using a differential shell momentum balance,


determine expressions for the steady-state shear
stress distribution and the velocity profile for a
Newtonian fluid (of viscosity μ).

b) Obtain expressions for the maximum velocity,


average velocity and the mass flow rate for slit flow.
• Rectangular Cartesian coordinates.
• Since the fluid flow is in the z-
direction, vx= 0, vy = 0, and only vz exists.
• vz is independent of z and it is meaningful
to postulate that velocity vz = vz(x) and
pressure p = p(z).
• The only nonvanishing components of the
stress tensor are τxz = τzx, which depend
only on x.
• Consider now a thin rectangular slab
(shell) perpendicular to the x-direction
extending a distance W in the y-direction
and a distance L in the z-direction.

Rate of z-momentum In − Out + Generation = Accumulation


• At steady-state, the accumulation term is zero.
• Momentum can go 'in' and 'out' of the shell by both the
convective and molecular mechanisms.
• Since vz(x) is the same at both ends of the slit, the
convective terms cancel out because (ρ vz vzW Δx)|z = 0 =
(ρ vz vz W Δx)|z = L.
• Only the molecular term (L W τxz ) remains to be
considered. Generation of z-momentum occurs by the
pressure force (p W Δx) and gravity force (ρ g W L Δx).
• On substituting these contributions into the z-momentum
balance, we get

(L W τxz ) | x − (L W τxz ) | x+Δx+ ( p 0 − p L ) W Δx + ρ g W L Δx = 0


τxz| x+Δx − τxz| x p 0 − p L + ρ g L
=
Δx L
• The general definition of the modified pressure is
P = p+ρgh,
where h is the distance upward (in the direction opposed to
gravity) from a reference plane of choice.
• Since the z-axis points downward in this problem, h= − z and
therefore P = p − ρ g z .
• Thus, P0 = p0 at z = 0 and PL = pL − ρ g L at z = L giving p0 − pL + ρ
g L = P0 − PL ≡ ΔP. Thus, it gives
𝑑𝜏𝑥𝑧 ∆𝑃
=
𝑑𝑥 𝐿
This 1st-order diferential equation may be simply integrated to give

C1 is determined later using boundary conditions.


• This equation applies to both Newtonian and non-Newtonian fluids,
and provide starting points for many fluid flow problems in rectangular
Cartesian coordinates
• Substituting Newton's law of viscosity for τxz

• After integration,

• The no-slip boundary conditions at the two fixed walls are

• 
• The final expression for the shear stress (or momentum flux)
distribution and velocity are found to be

• (i) The maximum velocity occurs at x = 0 (where dvz/dx = 0). Therefore

• (ii) The average velocity is obtained by dividing the volumetric flow rate
by the cross-sectional area as shown below.

• Thus, the ratio of the average velocity to the maximum velocity for
Newtonian fluid flow in a narrow slit is 2/3.
Non-Newtonian (Power law)
• For the region 0 ≤ x ≤ B, the velocity decreases with increasing x and
therefore dVz / dx ≤ 0. On the other hand, for the region −B ≤ x ≤ 0, the
velocity decreases with decreasing x and therefore dvz / dx ≥ 0. Thus,
the power law model is given as

• To obtain the velocity distribution for 0 ≤ x ≤ B, equations (12.a.2) and


the rst of (12.a.3) are combined to eliminate xz and get

• On integrating and using the no-slip boundary condition (vz = 0


at x = B ), we get
• Similarly, for −B ≤ x ≤ 0, equations (2) and (4) may be
combined to eliminate τxz and obtain

• Combining both,

• Note that the maximum velocity in the slit occurs at the


midplane x = 0.
Equations of change
for isothermal systems
Equations of Change for Isothermal Systems
• In the previous lecture, we saw how to derive the velocity
distribution for simple flows by the application of the shell
momentum balance or the force balance.
• It is however more reliable to start with general equations
for
o the conservation of mass (continuity equation)
o the conservation of momentum (eq, of motion, N2L)
to describe any flow problem and then simplify these
equations for the case at hand.
• For non-isothermal fluids (heat transfer + flow problems),
the same technique can be applied combined with the use
of the equation for the conservation of energy
Lagrangian vs. Eulerian description
A fluid flow field can be thought Another view of fluid motion is
of as being comprised of a large the Eulerian description. In the
number of finite sized fluid Eulerian description of fluid
particles which have mass, motion, we consider how flow
momentum, internal energy, and properties change at a fluid
other properties. Mathematical element that is fixed in space
laws can then be written for and time (x,y,z,t), rather than
each fluid particle. This is the following individual fluid
Lagrangian description of fluid particles.
motion.

Governing equations can be derived using each


method and converted 3to the other form.
Recall: Time Derivatives
• Consider any quantity c which depends on time as well as
position can be written as c=f(t, x, y, z) then its differential
dc is:

• Partial time derivative: This is the derivative of the


function c with time holding x, y, z
constant. (fixed observer)

• Total Time Derivative: This accounts for the fact that the
observer is moving (how c varies
with t because of changing location).
This is a specific case of the
• Substantial Time Derivative : Total Time Derivative for which the
velocity v of the observer is the
same as the velocity of the flow. It
is also called the Derivative
Following the Motion

• where the local fluyid velocity is defined by:


Rate of change for a fluid particle
• Terminology: fluid element is a volume stationary in space,
and a fluid particle is a volume of fluid moving with the flow.
• A moving fluid particle experiences two rates of changes:
o Change due to changes in the fluid as a function of time.
o Change due to the fact that it moves to a different location
in the fluid with different conditions.
• The sum of these two rates of changes for a property per unit
mass  is called the total or substantial derivative D /Dt:
D   dx  dy  dz
   
Dt t x dt y dt z dt
• With dx/dt=u, dy/dt=v, dz/dt=w, this results in:
D 
  v. grad 
Dt t
6
Fluid element and properties
• A fluid element can be thought of as the Fluid element for
smallest volume for which the continuum conservation laws
assumption is valid.
• Properties are averages of a sufficiently
large number of molecules Dz

• The behavior of the fluid is described in


terms of macroscopic properties: Dy
Dx
o Velocity v, Pressure p,Density r
z
y
x

7
Equation of Continuity
differential control volume:
Equation of Continuity
• Rate of Mass accumulation
= (Rate of Mass In) - (Rate of Mass Out)
• Rate of Mass In through face at x is

• Rate of Mass Out through face at x+Δx is


• For other faces, same as above
• Rate of Mass Accumulation is

Collecting the terms,


Equation of Continuity
• Dividing on both sides by ΔxΔyΔz and taking the limit as
ΔxΔyΔz goes to zero yields:

• This equation is known as the Continuity Equation (mass


balance for fixed observer).
Equation of Continuity
r    r vx    r v y    r vz  
    
t  x y z 
Can be written as, r  div (r V)  0
t
Net flow of mass across boundaries
Change in density

r r r r  vx v y vz 
 vx  vy  vz  r    
t x y z  x y z 

Dr  vx v y vz 
 r      r   v 
Dt  x y z 
Continuity equation
• For incompressible fluids r / t = 0, and the equation
becomes:
div v = 0.
vx v y vz ui
• Alternative ways to write this: x  y  z  0 and x  0
i

12
Different forms of the continuity equation

Finite control volume Finite control volume fixed


fixed in space mass moving with flow

 D
 r dV   r U  dS  0  r dV  0
t V S Dt V
Integral form Integral form
Conservation form Non  conservation form
U

Infinitesimally small Infinitesimally small fluid element of fixed


element fixed in space mass (“fluid particle”) moving with the flow

r Dr
   ( rU)  0  r  U  0
t Dt
Differential form Differential form
Conservation form 13
Non  conservation form
Differential Equation of Continuity In
cylindrical coordinates:
d r 1   r rvr  1   r v    r vz 
   0
dt r r r  z
 y
where r  x  y ,   tan  
2 2 1

x

If fluid is incompressible:
vr vr 1 v vz
   0
r r r  z
Equations of Motion
Fluid is flowing in 3
directions

For 1D fluid flow,


momentum transport
occurs in 3 directions

Momentum
transport is fully
defined by 3
equations of
motion
Momentum Balance
Consider the x-component of the momentum
transport:  Sum of forces 
 Rate of   Rate of   Rate of   
        acting in 
 accumulation  x  momentum in  x  momentum out  x  
 the system x

 Rate of   Rate of 
   
 momentum in x  momentum out x
 Rate of   Rate of  
     
 momentum in x  momentum out  x  convective
 Rate of   Rate of  
     
 momentum in x  momentum out  x  molecular
Momentum Balance

Due to convective transport:


 Rate of   Rate of  
    
 momentum in  x  momentum out  x  convective
  r vx vx  x   r vx vx  x Dx  DyDz

  r v y vx    r v y vx   DxDz
 y y Dy 

  r vz vx  z   r vz vx  z Dz  DxDy


Consider the x-component of the momentum
transport:

 Sum of forces 
 Rate of   Rate of   Rate of   
 
  
  
  acting in 
 accumulation x  momentum in x  momentum out x  
 the system  x

 Rate of   Rate of 
   
 momentum in x  momentum out x
 Rate of   Rate of  
     
 momentum in x  momentum out  x  convective
 Rate of   Rate of  
     
 momentum in x  momentum out  x  molecular
Due to convective transport:

 Rate of   Rate of  
    
 momentum in  x  momentum out  x  convective
  r vx vx  x   r vx vx  x Dx  DyDz

  r v y vx    r v y vx   DxDz
 y y Dy 

  r vz vx  z   r vz vx  z Dz  DxDy


Due to molecular transport:

 Rate of   Rate of  
    
 momentum in  x  momentum out  x  molecular
  xx  x   xx  x Dx  DyDz

  yx    yx   DxDz
 y y  Dy 

  zx  z   zx  z Dz  DxDy


Consider the x-component of the momentum
transport:  Sum of forces 
 Rate of   Rate of   Rate of   
        acting in 
 accumulation  x  momentum in  x  momentum out  x  
 the system x

 Sum of forces 
 
 acting in    px  px Dx  DyDz  r g x DxDyDz
 the system 
 x
Consider the x-component of the momentum
transport:
 Sum of forces 
 Rate of   Rate of   Rate of   
 
  
  
  acting in 
 accumulation x  momentum in x  momentum out x  
 the system  x

 Rate of    r vx 
   DxDyDz
 accumulation  x t
Substituting:
  r vx 
DxDyDz   r vx vx  x   r vx vx  x Dx  DyDz
t
  r v y v x    r v y v x   DxDz
 y y  Dy 

  r vz vx  z   r vz vx  z Dz  DxDy


  xx  x   xx  x  Dx  DyDz

  yx    yx   DxDz
 y y Dy 

  zx  z   zx  z Dz  DxDy


  px  px Dx  DyDz  r g x DxDyDz
Dividing everything by ΔV:

  r vx   r vx vx  x   r vx vx  x Dx   r v y vx  y   r v y vx  y Dy 


 
t Dx Dy
 r vz vx  z   r vz vx  z  Dz   xx  x   xx  x Dx 
 
Dz Dx
      
 yx y yx y  Dy
   zx  z   zx  z Dz 
 
Dy Dz


 px  px Dx 
 rg
Dx
x
Taking the limit as ∆x, ∆y and ∆z  0:

  r vx    r vx vx    r v y vx    r v z v x 
  
t x y z
  xx    yx    zx  p
     r gx
x y z x
Rearranging:
  r vx    r vx vx    r v y vx    r vz vx 
  
t x y z
   xx    yx    zx   p
      r gx
 x y z  x

For the convective terms:
  r vx vx    r v y vx    r vz vx 
 
x y z
 vx vx vx     r vx    r v y    r vz  
 r  vx  vy  vz   vx    
 x y z   x y z 
For the accumulation term:
  r vx  vx r
r  vx
t t t
vx   vx v y vz   r r r  
r  vx  r       vx  vy  vz 
t   x y z   x y z  
Substituting:

vx   vx v y vz   r r r  


r  vx  r       vx  vy  vz 
t   x y z   x y z  
 vx vx vx 
 r  vx  vy  vz 
 x y z 
   r vx    r v y    r vz  
 vx    
 x y z 
   xx    yx    zx   p
      r gx
 x y z  x

Substituting:
vx  vx vx vx 
r  r  vx  vy  vz 
t  x y z 
   xx    yx    zx   p
      r gx
 x y z  x

EQUATION OF MOTION FOR THE x-
COMPONENT
Substituting:
v y  v y v y v y 
r  r  vx  vy  vz 
t  x y z 
   xy    yy    zy   p
      rgy
 x y z  y

EQUATION OF MOTION FOR THE y-
COMPONENT
Substituting:
vz  vz vz vz 
r  r  vx  vy  vz 
t  x y z 
   xz    yz    zz   p
      r gz
 x y z  z

EQUATION OF MOTION FOR THE z-


COMPONENT
Differential Equation of Motion

vx  vx vx vx     xx    yx    zx   p


r  r  vx  vy  vz        r gx
t  x y z   x y z  x

v y  v y v y v y     xy    yy    zy   p
r  r  vx  vy  vz         r gy
t  x y z   x y z  y

vz  vz vz vz     xz    yz    zz   p


r  r  vx  vy  vz         r gz
t  x y z   x y z  z

Differential Equation of Motion
Dvx    xx    yx    zx   p
r       r gx
Dt  x y z  x

Dv y    xy    yy    zy   p
r       rgy
Dt  x y z  y

Dvz    xz    yz    zz   p
r       r gz
Dt  x y z  z

Dv
r        p  r g
Dt
Navier-Stokes Equations

Assumptions
1. Newtonian fluid
2. Obeys Stokes’ hypothesis *
3. Continuum
4. Isotropic viscosity
5. Constant density
Navier-Stokes Equations
Dvx p   2 vx  2 vx  2 vx 
r    r gx    2  2  2 
Dt x  x y z 

Dv y p   vy  vy  vy
2 2 2

r    r gy    2  2  2
 x 
Dt y  y z 

Dvz p   2 vz  2 vz  2 vz 
r    r gz    2  2  2 
Dt z  x y z 

Dv
r  p  r g   2 v
Dt
Navier-Stokes Equations
 vr vr v vr v2 vr 
r  vr    vz 
  t r r  r z 
p    1   rvr   1  2 vr 2  vr   vr 
2
   r gr      2  2  2 
r  r  r r  r  r    z 
2

 vr vr v vr v2 v 


r  vr    vz r 
 t r r  r z 
1 p    1   rv   1  2 v 2  v   v
2

  r g      2  2  2 
r   r  r r  r 
2
r    z 
 vr vr v vr v2 vr 
r  vr    vz 
  t r r  r z 
p  1   vz  1  vz  vz 
2 2
   r gz    r  2  2 
z  r r  r  r  z 
2
Application
The Navier-Stokes equations may be reduced
using the following simplifying assumptions:
1. Steady state flow
vx  vx vx vx  p   2 vx  2 vx  2 vx 
r  r  vx  vy  vz     r gx    2  2  2 
t  x y z  x  x y z 
v y  v y v y v y  p   2v y  2v y  2v y 
r  r  vx  vy  vz     r g y    2  2  2 
t  x y z  y  x y z 
vz  vz vz vz  p   2 vz  2 vz  2 vz 
r  r  vx  vy  vz     r gz    2  2  2 
t  x y z  z  x y z 
Application

The Navier-Stokes equations may be reduced


using the following simplifying assumptions:
2. Unidirectional flow
 vx vx vx  p   2 vx  2 vx  2 vx 
r  vx  vy  vz     r gx    2  2  2 
 x y z  x  x y z 
 v y v y v y  p   2v y  2v y  2v y 
r  vx  vy  vz     r g y    2  2  2 
 x y z  y  x y z 
 vz vz vz  p   2 v z  2 v z  2 vz 
r  vx  vy  vz     r gz    2  2  2 
 x y z  z  x y z 
Application
The Navier-Stokes equations may be reduced
using the following simplifying assumptions:
3. No viscous dissipation (INVISCID FLOW)
vx p   2 vx  2 vx  2 vx 
r vx    r gx    2  2  2 
x x  x y z 
0  0  r gy vx p
r vx    r gx
0  0  r gz x x
Euler’s equation

Integration of the Euler’s equation along a stream line gives the Bernoulli eqn.
Application

The Navier-Stokes equations may be reduced


using the following simplifying assumptions:
4. No external forces acting on the system
Inviscid flow: vx
r vx 0
x
vx
 0  vx  constant
x
Application
The Navier-Stokes equations may be reduced
using the following simplifying assumptions:
4. No external forces acting on the system

Viscous flow:

vx   vx  vx  vx 
2 2 2
r vx   2  2  2 
x  x y z 
Application
The Navier-Stokes equations may be reduced
using the following simplifying assumptions:
5. Semi-infinite system

Dy Dx, Dz
Dz
Dy
Dx
vx   vx  vx  vx 
2 2 2
r vx   2  2  2 
x  x y z 
Application
The Navier-Stokes equations may be reduced
using the following simplifying assumptions:
6. Laminar flow (no convective transport)

vx  vx2
r vx  2
x y
 vx 2
 2 0  vx  c1 y  c2
y
Application
The Navier-Stokes equations may be reduced
using the following simplifying assumptions:
6. Laminar flow (no convective transport)
Boundary conditions:
1 at y  0  lower plate  , vx  v0 
 v0
 c2  v0 , c1  
 2 at y    upper plate  , vx  0  

v0  y
vx   y  v0 or vx  v0 1  
  
Example
 Example 1 (bent pipe), page 47
r1,V1
Bx y

Area=A1 Area=A2=A1 x


By
r2,V2
Fx  P1 A1  P2 A2 Cos( )  Bx

Fy  0  P2 A2 Sin( )  W  By
Weight of Fluid
Example

Flux X  V2 Cos( ) r2V2 A2   V1 r1V1 A1 

out in

FluxY  0   V2 Sin( ) r2V2 A2 


in

 Steady State out


 d/dt =0
 From Eqn of Conservation of Mass
r1V1 A1  r2V2 A2  m
Example

 F   V r V . n dA  t  V r d (Vol )
s Vol

V1
By

Area=A1 Area=A2=A1

Q
Bx
V2

 V1  V2Cos( )
 Bx  P1 A1  P2 A2 Cos( )  m

 By  P2 A2 Sin( )  W  m
 V2 sin( )
Example
P1,V1  Pipe with U turn

P2,V2

Total Force  F  P1 A1  P2 A2 Use Gage Pressure!

In case of gas, use


Flux   rV A1  rV A2
1
2
2
2 absolute pressure to
calculate density
Example N

P1  “L” bend
E
F

Total Force x  Fx  P1 A1 Assume the force by the pipe on


the fluid is in the positive

Total Force y  Fy  P2 A2
direction
P2

Fluxx   rV A1
1
2

What will the force be, if the flow is reversed


Fluxy   rV A2
2
2
(a) in a straight pipe? (b) in a L bend?
Application of the
Momentum Equation
• Application of the Momentum Equation

We will consider the following examples:

1. Force due to the flow of fluid round a pipe bend.

2. Force on a nozzle at the outlet of a pipe.


The force due the flow around a pipe bend

Consider a pipe bend with a constant cross


section lying in the horizontal plane and turning
through an angle of θ
• Because the fluid changes direction, a force (very large in
the case of water supply pipes,) will act in the bend. If the
bend is not fixed it will move and eventually break at the
joints. We need to know how much force a support (thrust
block) must withstand.
• Step in Analysis:
1. Draw a control volume
2. Decide on co-ordinate axis system
3. Calculate the total force (rate of change of
momentum)
4. Calculate the pressure force
5. Calculate the body force
6. Calculate the resultant force
• The control volume is draw in the above figure, with faces
at the inlet and outlet of the bend and encompassing the
pipe walls.
• It is convenient to choose the co-ordinate axis so that one
is pointing in the direction of the inlet velocity.
• In the above figure the x-axis points in the direction of the
inlet velocity.
• Calculate the total force:
In the x-direction:

Ftx  Q(u2 x  u1x )


u1x  u1
u2 x  u2 cos 
Ftx  Q(u2 cos   u1 )
In the y-direction:

Fty  Q(u2 y  u1 y )
u1 y  u1 sin 0  0
u2 y  u2 sin 
Fty  Qu2 sin 
• Calculate the pressure force
F p = pressure force at 1 - pressure force at 2
Fpx = p1 A1 cos 0 – p2 A2 cos θ = p1 A1 – p2 A2 cos θ
Fpy = p1 A1 sin 0 – p2 A2 sin θ = – p2 A2 sin θ

• Calculate the body force


There are no body forces in the x or y directions. The only
body force is that exerted by gravity (which acts into the
paper in this example - a direction we do not need to
consider).
Fbx = Fby = 0
• Calculate the resultant force
FRx = Ftx - Fpx - Fbx
FRy = Fty - Fpy - Fby

FRx  FTx  Fpx  0  Q(u2 cos   u1 )  p1 A1  p2 A2 cos 


FRy  FTy  Fpy  0  Qu2 sin   p2 A2 sin 
And

Fresultan t  FRx2  FRy2


FRy
  tan (1
)
FRx
The force on the bend is the same magnitude but in the
opposite direction

R = - Fresultant
Force on a pipe nozzle
Force on the nozzle at the outlet of a pipe. Because
the fluid is contracted at the nozzle forces are
induced in the nozzle. Anything holding the nozzle
(e.g. a fireman) must be strong enough to withstand
these forces.
• The same analysis will be followed as before.

• Step in Analysis:
1. Draw a control volume
2. Decide on co-ordinate axis system
3. Calculate the total force
4. Calculate the pressure force
5. Calculate the body force
6. Calculate the resultant force
• 1 & 2 Control volume and Co-ordinate axis are shown in
the figure above.
• 3 Calculate the total force

FT  FTx  Q(u2  u1 )
Using the continuity, Q = A u

1 1
FTx  Q (  )
2

A2 A1
• 4 Calculate the pressure force

Fp = Fpx = pressure force at 1 - pressure force at 2

p1 u12 p2 u22
  z1    gz2  h f
g 2 g g 2 g
Friction is neglected for short distance and the nozzle is
horizontal i.e. z1 and z2 are equal and p2 is atmospheric
i.e. 0 and with continuity

Q 2 1 1
p1  ( 2  2)
2 A2 A1
• 5 Calculate the body force
The only body force is the weight due to gravity in the y-
direction - but we need not consider this as the
only forces we are considering are in the x-direction.

• 6 Calculate the resultant force

FRx  FTx  Fpx  FBx


FRx  FTx  Fpx  0
1 1 Q 2
1 1
FTx  Q (  ) 
2
( 2  2)
A2 A1 2 A2 A1
• So the fireman must be able to resist the force of

R   FRx
Example
 Example 1 (bent pipe), page 47
1,V1
Bx y

Area=A1 Area=A2=A1 x


By
2,V2
Fx  P1 A1  P2 A2 Cos( )  Bx

Fy  0  P2 A2 Sin( )  W  By
Weight of Fluid
Example

Flux X  V2 Cos( ) 2V2 A2   V1 1V1 A1 

out in

FluxY  0   V2 Sin( ) 2V2 A2 


in

 Steady State out


 d/dt =0
 From Eqn of Conservation of Mass
1V1 A1  2V2 A2  m
Example

 F   V  V . n dA  t  V  d (Vol )
s Vol

V1
By

Area=A1 Area=A2=A1

Q
Bx
V2

 V1  V2Cos( )
 Bx  P1 A1  P2 A2 Cos( )  m

 By  P2 A2 Sin( )  W  m
 V2 sin( )
Example
P1,V1  Pipe with U turn

P2,V2

Total Force  F  P1 A1  P2 A2 Use Gage Pressure!

In case of gas, use


Flux   V A1  V A2
1
2
2
2 absolute pressure to
calculate density
Example N

P1  “L” bend
E
F

Total Force x  Fx  P1 A1 Assume the force by the pipe on


the fluid is in the positive

Total Force y  Fy  P2 A2
direction
P2

Fluxx   V A1
1
2

What will the force be, if the flow is reversed


Fluxy   V A2
2
2
(a) in a straight pipe? (b) in a L bend?
Energy equation
Energy equation
• First law of thermodynamics: rate of change of energy of a
fluid particle is equal to the rate of heat addition plus the
rate of work done.
• Rate of increase of energy is DE/Dt.
• Energy E = i + ½ (u2+v2+w2).
• Here, i is the internal (thermal energy).
• ½ (u2+v2+w2) is the kinetic energy.
• Potential energy (gravitation) is usually treated separately
and included as a source term.
• We will derive the energy equation by setting the total
derivative equal to the change in energy as a result of work
done by viscous stresses and the net heat conduction.

22
Work done by surface stresses in x-direction

v x zx |z  z xy
v x yx |x xz v x yx |x  x xz

v x p |x yz v x p |x  x yz

v x xx |x yz v x xx |x  x yz

z
y v x zx |z xy
x
Work done is force times velocity. 23
Work done by surface stresses
• The total rate of work done by surface stresses is
calculated as follows:
o For work done by x-components of stresses add all
terms in the previous slide.
o Do the same for the y- and z-components.
• Add all and divide by xyz to get the work done per unit
volume by the surface stresses:

 ( v x xx )  ( v x yx )  ( v x zx )  ( v y xy )
 div ( pu)    
x y z x
 ( v y yy )  ( v y zy )  ( v z xz )  ( vz yz )  ( v z zz )
    
y z x y z

24
Energy flux due to heat conduction
qz |z  z xy
q y |y  y xz

qx |x yz
qx |x  x yz

z
y
x

qz |z xy q y |y xz

The heat flux vector q has three components, qx, qy, and qz.
25
Energy flux due to heat conduction
• Summing all terms and dividing by xyz gives the net rate of
heat transfer to the fluid particle per unit volume:
q x q y q z
     div q
x y z
• Fourier’s law of heat conduction relates the heat flux to the
local temperature gradient:
T T T
qx   k qy   k qz   k
x y z
• In vector form: q   k grad T
• Thus, energy flux due to conduction:  div q  div(k grad T )
• This is the final form used in the energy equation.
26
Energy equation
• Setting the total derivative for the energy in a fluid particle
equal to the previously derived work and energy flux terms,
results in the following energy equation:
DE   ( v x xx )  ( v x yx )  ( v x zx )  ( v y xy )
   div ( pu)     
Dt  x y z x
 ( v y yy )  ( v y zy )  ( v z xz )  ( v z yz )  ( v z zz ) 
    
y z x y z 
 div ( k grad T )  S E

• Note that we also added a source term SE that includes


sources (potential energy, sources due to heat production
from chemical reactions, etc.).
27
Kinetic energy equation
• Separately, we can derive a conservation equation for the
kinetic energy of the fluid.
• In order to do this, we multiply the u-momentum equation
by u, the v-momentum equation by v, and the w-
momentum equation by w. We then add the results
together.
• This results in the following equation for the kinetic energy:

D[ 12 (u 2  v 2  w 2 )]   xx  yx  zx 
   u.grad p  u   
Dt  x y z 
  xy  yy  zy    xz  yz  zz 
 v     w     u. S M
 x y z   x y z 

28
Internal energy equation
• Subtract the kinetic energy equation from the energy
equation.
• Define a new source term for the internal energy as
Si = SE - u.SM. This results in:

Di  u u u v
   p div u   xx   yx   zx   xy
Dt  x y z x
v v w w u 
  yy   zy   xz   yz   zz 
y z x y z 
 div (k grad T )  S i

29
Enthalpy equation
• An often used alternative form of the energy equation is
the total enthalpy equation.
o Specific enthalpy h = i + p/.
o Total enthalpy h0 = h + ½ (u2+v2+w2) = E + p/.

 (  h0 )
 div(  h0u)  div(k grad T )
t
  (u xx )  (u yx )  (u zx )  (v xy )
    
 x y z x
 (v yy )  (v zy )  ( w xz )  ( w yz )  (u zz ) 
    
y z x y z 
 Sh
30
Equations of state
• Fluid motion is described by five partial differential
equations for mass, momentum, and energy.
• Amongst the unknowns are four thermodynamic variables:
, p, i, and T.
• We will assume thermodynamic equilibrium, i.e. that the
time it takes for a fluid particle to adjust to new conditions
is short relative to the timescale of the flow.
• We add two equations of state using the two state
variables  and T: p=p(,T) and i=i(,T).
• For a perfect gas, these become: p= RT and i=CvT.
• At low speeds (e.g. Ma < 0.2), the fluids can be considered
incompressible. There is no linkage between the energy
equation, and the mass and momentum equation. We then
only need to solve for energy if the problem involves heat
transfer. 31
Viscous stresses
• A model for the viscous stresses ij is required.
• We will express the viscous stresses as functions of the
local deformation rate (strain rate) tensor.
• There are two types of deformation:
o Linear deformation rates due to velocity gradients.
• Elongating stress components (stretching).
• Shearing stress components.
o Volumetric deformation rates due to expansion or
compression.
• All gases and most fluids are isotropic: viscosity is a scalar.
• Some fluids have anisotropic viscous stress properties,
such as certain polymers and dough. We will not discuss
those here.
32
Heat Transfer
Mekonnen Gebrehiwot
Objectives
• Understand the basic mechanisms of heat transfer,
• Identify the mechanisms of heat transfer that occur
simultaneously in practice
• Develop an awareness of the cost associated with
heat losses
• Solve various heat transfer problems encountered in
practice

2
HEAT TRANSFER
• Heat: The form of energy that can be transferred from one
system to another as a result of temperature difference.
• Heat Transfer deals with the determination of the rates of
such energy transfers as well as variation of temperature.
• The transfer of energy as heat is always from the higher-
temperature medium to the lower-temperature one.
• Heat transfer stops when the two mediums reach the same
temperature.
• Heat can be transferred in three different modes:
conduction, convection, radiation

3
Heat Transfer supplements the First and Second Laws of Thermodynamics,
with additional rules to analyse the Energy Transfer RATES.
4
Application Areas of Heat Transfer

5
Energy Transfer
Energy can be transferred to or from a given mass
by two mechanisms: heat transfer and work. when is constant:
Heat transfer rate: The amount of heat transferred
per unit time.
Heat flux: The rate of heat transfer per unit area
normal to the direction of heat transfer.

Power: The work


done per unit time.

6
HEAT TRANSFER MECHANISMS
• Heat can be transferred in three basic modes:
o conduction

o convection

o Radiation

• All modes of heat transfer require the existence of a temperature


difference.
• The transfer of energy as heat is always from the higher-temperature
medium to the lower-temperature one, and heat transfer stops when
the two mediums reach the same temperature.
7
Conduction, Convection & Thermal Radiation
• Conduction refers to the transport of energy in a medium due to a
temperature gradient.
• In contrast, the convection refers to heat transfer that occurs
between a surface and a fluid (at rest or in motion) when they are at
different temperatures.

• Thermal radiation refers to


the heat transfer that occurs
between two surfaces at
different temperatures
emitted/absorbed by any
surface in the form of
electromagnetic waves.
CONDUCTION
Conduction: The transfer of energy from the more energetic particles of a
substance to the adjacent less energetic ones as a result of interactions
between the particles.
In gases and liquids, conduction is due to the collisions and diffusion of the
molecules during their random motion.
In solids, it is due to the combination of vibrations of the molecules in a lattice
and the energy transport by free electrons.
The rate of heat conduction through a plane layer is
proportional to the temperature difference across the
layer and the heat transfer area, but is inversely
proportional to the thickness of the layer.

In heat conduction analysis,


A represents the area Heat conduction
normal to the direction of through a large plane
heat transfer. wall of thickness x
and area A.
10
When x → 0
Fourier’s law of heat
conduction

Thermal conductivity, k: A measure of the ability of


a material to conduct heat.

Temperature gradient dT/dx: The slope of the


temperature curve on a T-x diagram.
Heat is conducted in the direction of decreasing
temperature, and the temperature gradient becomes
negative when temperature decreases with
increasing x.
The negative sign in the equation ensures that heat
transfer in the positive x direction is a positive
quantity.

The rate of heat conduction through a solid is


directly proportional to its thermal conductivity.
11
12
Thermal Conductivity
• Thermal conductivity:
The rate of heat transfer
through a unit thickness
of the material per unit
area per unit temperature
difference.

• The thermal conductivity


of a material is a measure
of the ability of the
material to conduct heat.

• A high value for thermal


conductivity indicates that
the material is a good
heat conductor, and a low A simple experimental setup
value indicates that the to determine the thermal
material is a poor heat conductivity of a material.
conductor or insulator. 13
The range of thermal conductivity of various materials at room temperature.
14
The variation of the thermal conductivity of various solids, liquids, and gases with
temperature.

15
Thermal Diffusivity
cp Specific heat, J/kg · °C: Heat capacity
per unit mass
cp Heat capacity, J/m3·°C: Heat capacity
per unit volume
 Thermal diffusivity, m2/s: Represents
how fast heat diffuses through a material

• A material that has a high thermal


conductivity or a low heat capacity will
obviously have a large thermal diffusivity.
• The larger the thermal diffusivity, the faster
the propagation of heat into the medium.
• A small value of thermal diffusivity means
that heat is mostly absorbed by the
material and a small amount of heat is
16
conducted further.
Example:
• The Cost of Heat Loss through a Roof
The roof of an electrically heated home is 6 m long, 8 m wide, and 0.25
m thick, and is made of a flat layer of concrete whose thermal
conductivity is k=0.8 W/m · °C . The temperatures of the inner and the
outer surfaces of the roof one night are measured to be 25°C and 0°C,
respectively, for a period of 10 hours.
Determine:
(a) the rate of heat loss through the roof that night and
(b) the cost of that heat loss to the home owner if the cost of electricity is
$0.2/kWh.

0 °C

25 °C
Solution:

(a) Noting that heat transfer through the roof is by conduction and the
area of the roof is:
A= 6 m×8 m=48 m2.
The steady rate of heat transfer through the roof is determined to be

𝑄=kA(T 1-T2)/L= (0.8)(48 )(25-0)/0.25= 3840 W= 3.84 kW

(b) The amount of heat lost through the roof during a 10-hour period and
its cost are determined from
Q = 𝑄tሶ =(3.84 kW)(10 h) = 38.4 kWh Why is the
Cost/day = (Amount of energy)(Unit cost of energy) bill so high?
= (38.4 kWh)($0.2/kWh) =$7.68
Cost/month = (cost/day)×(30day/month)= $7.68×30=$230.4
CONVECTION
• Convection: The mode of
energy transfer between a
solid surface and the
adjacent liquid or gas that
is in motion, and it involves
the combined effects of
conduction and fluid
motion.
• The faster the fluid motion,
the greater the convection
heat transfer.
• In the absence of any bulk
fluid motion, heat transfer
Heat transfer from a hot surface to air
between a solid surface
by convection.
and the adjacent fluid is by
pure conduction. 19
• Forced convection: If the fluid
is forced to flow over the
surface by external means
such as a fan, pump, or the
wind.
• Natural (or free) convection:
If the fluid motion is caused by
buoyancy forces that are
induced by density differences
due to the variation of The cooling of a boiled egg by
temperature in the fluid. forced and natural convection.

Heat transfer processes that involve change of phase of a fluid are also
considered to be convection because of the fluid motion induced during
the process, such as the rise of the vapor bubbles during boiling or the
fall of the liquid droplets during condensation.

20
Newton’s law of cooling

h convection heat transfer coefficient, W/m2 · °C


As the surface area through which convection heat transfer takes place
Ts the surface temperature
T the temperature of the fluid sufficiently far from the surface.

• The convection heat transfer


coefficient h is not a property of
the fluid.
• It is an experimentally determined
parameter whose value depends
on all the variables influencing
convection such as
o the surface geometry
o the nature of fluid motion
o the properties of the fluid
o the bulk fluid velocity
21
22
RADIATION

• Radiation: The energy emitted by matter in the form of electromagnetic


waves (or photons) as a result of the changes in the electronic
configurations of the atoms or molecules.
• Unlike conduction and convection, the transfer of heat by radiation does
not require the presence of an intervening medium.
• Heat transfer by radiation is fastest (at the speed of light). It suffers no
attenuation in a vacuum.
• This is how the energy of the sun reaches the earth.
23
• In heat transfer studies we are interested in thermal radiation, which is
the form of radiation emitted by bodies because of their temperature.
• All bodies at a temperature above absolute zero emit thermal radiation.
• Radiation is a volumetric phenomenon, and all solids, liquids, and
gases emit, absorb, or transmit radiation to varying degrees.
• However, radiation is usually considered to be a surface phenomenon
for solids opaque to thermal radiation such as metals, wood, and rocks .
• The maximum rate of radiation that can be emitted from a surface at an
absolute temperature Ts is given by the Stefan–Boltzmann law as

Stefan–Boltzmann law
 = 5.670  108 W/m2 · K4 Stefan–Boltzmann constant

Blackbody: The idealized surface that emits radiation at the maximum rate.

24
Radiation emitted by real surfaces

Emissivity  : A measure of how closely


a surface approximates a blackbody for
which  = 1 of the surface. 0   1.

• Blackbody radiation represents the


maximum amount of radiation that can
be emitted from a surface at a
specified temperature. 25
Absorptivity : The fraction of the radiation energy incident on a
surface that is absorbed by the surface. 0   1
A blackbody absorbs the entire radiation incident on it ( = 1).
Kirchhoff’s law: The emissivity and the absorptivity of a surface at
a given temperature and wavelength are equal.

The absorption of radiation incident on an opaque surface of absorptivity.


26
• Net radiation heat transfer: The difference between the rates of
radiation emitted by the surface and the radiation absorbed.

• If the rate of radiation absorption is greater than the rate of radiation


emission, the surface is said to be gaining energy by radiation.
Otherwise, the surface is said to be losing energy by radiation.

• The determination of the net rate of heat transfer by radiation between


two surfaces is a complicated matter since it depends on
• the properties of the surfaces
• their orientation relative to each other
• the interaction of the medium between the surfaces with radiation

Radiation is usually significant relative to conduction or natural


convection, but negligible relative to forced convection.

27
When a surface is completely enclosed by a much larger (or black)
surface at temperature Tsurr separated by a gas (such as air) that does
not intervene with radiation, the net rate of radiation heat transfer
between these two surfaces is given by

Radiation heat transfer between a


surface and the surfaces surrounding it.

28
Summary
Heat Transfer Mechanisms
Conduction
Fourier’s law of heat conduction
Thermal Conductivity
Thermal Diffusivity
Convection
Newton’s law of cooling
Radiation
Stefan–Boltzmann law
Summary

• Conduction & convection require a temperature difference across a medium


(the interactions of atoms/molecules)
• Radiation transport can occur across a vacuum
SIMULTANEOUS HEAT
TRANSFER MECHANISMS
Heat transfer is only by conduction in opaque solids, but by
conduction and radiation in semitransparent solids.

Heat transfer is by conduction and possibly by radiation in a


still fluid (no bulk fluid motion) and by convection and
radiation in a flowing fluid.

In the absence of radiation, heat transfer through a fluid is


either by conduction or convection, depending on the
presence of any bulk fluid motion.
Convection = Conduction + Fluid motion
Heat transfer through a vacuum is by radiation.

Most gases between two solid surfaces Although there are three mechanisms of heat
do not interfere with radiation. transfer, a medium may involve only two of
them simultaneously.
Liquids are usually strong absorbers of
31
radiation.
When radiation and convection occur simultaneously between a
surface and a gas:

Combined heat transfer coefficient hcombined


Includes the effects of both convection and radiation

32
Thermal Resistance

Δx

R is called thermal
resistance
33
• Similarly for convection,

• For Radiation,

34
Combined Thermal Resistance
• Often, Convection and Radiation act together
o The resistances of convection and radiation are considered as a
single resistance:

o Using this concept, one can construct complete thermal circuits of


resistances in parallel and in series

For the heat transfer through a


plane wall:

35
Combined Thermal Resistance
• The total thermal resistance is given by,

• If the object considered has different layers, one can easily simulate
this by introducing the appropriate thermal resistances.
• one can also introduce contact resistances between different layers.

36
Multilayered plane walls

37
Parallel Walls
• Uniform Temperature at 1 and 2
• Heat flux split between sections
• Thermal Resistance based on local
areas
• Set up system of equations based upon
energy balance into nodes
39
40
Conduction in Cylinders and spheres
Consider steady heat conduction through a hot-water pipe.
• Heat transfer happens in the normal direction to the pipe surface.
• Fluid temperatures inside and outside the pipe remain constant,
o Thus heat transfer modeled as steady and one-dimensional.

• T is independent of azimuthal angle or axial distance

43
• Thus heat transfer modeled as steady and one-dimensional:

• The general resistance is then,

• For a cylinder,
2𝜋𝑟𝐿

• For a sphere,

44
Critical radius of insulation
• We know
o Adding more insulation to a plane wall decreases heat transfer
o The thicker the insulation, the lower the heat transfer rate
o This is expected, since the heat transfer area A is constant, and
adding insulation always increases the thermal resistance of
the wall without increasing the convection resistance.

• In a cylindrical pipe or a spherical shell,


o it increases the conduction resistance of the insulation layer
o but decreases the convection resistance of the surface
because of the increase in the outer surface area for
convection.
45
 The heat transfer from the pipe may increase or
decrease, depending on which effect dominates.

An insulated cylindrical pipe exposed to convection from the outer


surface and the thermal resistance network associated with it.

46
The critical radius of insulation for a cylindrical body:

The critical radius of insulation


for a spherical shell:

The largest value of the critical


radius we are likely to
encounter is

We can insulate hot-water or


The variation of heat transfer rate with the
steam pipes freely without
outer radius of the insulation r2 when r1 < rcr.
worrying about the possibility of
increasing the heat transfer by 47
insulating the pipes.
Heat conduction analysis
(1d and 3d)

II-48
Conduction
• Fourier’s Law 

q : Heat transfer rate

: Temperature gradient in direction of the heat flow

II-49
Heat Conduction Analysis- 1D
• 1-D (plane wall)

• So, the energy balance

II-50
II-51
Heat Conduction Analysis- 1D

General 1-Dimension Heat Conduction equation:

II-52
Heat Conduction Analysis-3D
1) Cartesian coordinates

• So, the energy balance:

II-53
1) Cartesian coordinates

II-54
Heat Conduction Analysis- 3D
General 3-Dimension Heat Conduction equation:
1) Cartesian coordinates

If, k = constant

: Thermal Diffusivity of the material

II-55
Thermal Diffusivity
• The larger value of α, the faster heat will diffuse through
the material.

• So, α can be higher value if


i) Thermal conductivity, K  higher
ii) Thermal heat capacity, ρc  lower

II-56
Heat Conduction Analysis- 3D
3-Dimension Heat Conduction equation:
2) Cylindrical coordinates

3) Spherical coordinates

II-57
Convection Heat Transfer

• Heat transfer by convection is more difficult to


analyze than heat transfer by conduction .

• No single property of the heat transfer medium,


such as thermal conductivity, can be defined to
describe the mechanism.

II-58
• Heat transfer by convection varies from situation
to situation (upon the fluid flow conditions), and it
is frequently coupled with the mode of fluid flow.
• In practice, analysis of heat transfer by
convection is treated empirically (by direct
observation).
• Convection heat transfer is treated empirically
because of several factors:
o Fluid velocity
o Fluid viscosity
o Heat flux
o Surface roughness
o Type of flow (single-phase/two-phase)

II-59
Convection
• Heat transfer in the presence of a fluid motion on a solid surface
•Various mechanisms at play in the fluid:
- advection  physical transport of the fluid
- diffusion  conduction in the fluid
- generation  due to fluid friction
•But fluid directly in contact with the wall does not move relative to it; hence
direct heat transport to the fluid is by conduction in the fluid only.

U y U T
q y
u(y) T(y)
Ts
y T
T  U
qconv   kf   h Ts  T  T(y)
y  y 0
T 
Ts

 depends
But y  y 0 on the whole fluid motion, and both fluid flow
and heat transfer equations are needed
Convection
Free or natural convection
(induced by buoyancy May occur with
forces) phase change
Convection (boiling,
condensation)
forced convection (driven
externally)

Heat transfer rate q = h( Ts-T  )W


Typical values of h (W/m2K)

h=heat transfer coefficient (W /m2K) Free convection: gases: 2 - 25


(h is not a property. It depends on liquid: 50 - 100
geometry ,nature of flow,
thermodynamics properties etc.)
Forced convection: gases: 25 - 250
liquid: 50 - 20,000
Boiling/Condensation: 2500 -100,000
Convection rate equation
U U T
y y
u(y) q T(y)
Ts
Main purpose of convective heat q=heat flux = h(Ts - T)
transfer analysis is to determine:
q = -k(T/ y)y=0
• flow field
Hence, h = [-k(T/ y)y=0] / (Ts - T)
• temperature field in fluid
• heat transfer coefficient, h

The expression shows that in order to determine h, we


must first determine the temperature distribution in the
thin fluid layer that coats the wall.
Classes of convective flows:
• extremely diverse
• several parameters involved (fluid properties, geometry, nature of flow,
phases etc)
• systematic approach required
• classify flows into certain types, based on certain parameters
• identify parameters governing the flow, and group them into meaningful
non-dimensional numbers
• need to understand the physics behind each phenomenon

Common classifications:
A. Based on geometry:
External flow / Internal flow
B. Based on driving mechanism
Natural convection / forced convection / mixed convection
C. Based on number of phases
Single phase / multiple phase
D. Based on nature of flow
Laminar / turbulent
How to solve a convection problem ?
• Solve governing equations along with boundary conditions
• Governing equations include
1. conservation of mass
2. conservation of momentum
3. conservation of energy
• In Conduction problems, only (3) is needed to be solved.
Hence, only few parameters are involved
• In Convection, all the governing equations need to be
solved.
 large number of parameters can be involved
Forced convection: Non-dimensional groupings
• Nusselt No. Nu = hx / k = (convection heat transfer strength)/
(conduction heat transfer strength)
• Prandtl No. Pr = / = (momentum diffusivity)/ (thermal diffusivity)
• Reynolds No. Re = U x /  = (inertia force)/(viscous force)
Viscous force provides the dampening effect for disturbances in the
fluid. If dampening is strong enough  laminar flow
Otherwise, instability  turbulent flow  critical Reynolds number

d d

Laminar Turbulent
FORCED CONVECTION:
external flow (over flat plate)
An internal flow is surrounded by solid boundaries that can restrict the
development of its boundary layer, for example, a pipe flow. An external flow, on
the other hand, are flows over bodies immersed in an unbounded fluid so that the
flow boundary layer can grow freely in one direction. Examples include the flows
over airfoils, ship hulls, turbine blades, etc.
bles •Fluid particle adjacent to the
solid surface is at rest
T

Ts
•These particles act to retard the
motion of adjoining layers
x q
• boundary layer effect
Momentum balance: inertia forces, pressure gradient, viscous forces,
body forces
Energy balance: convective flux, diffusive flux, heat generation, energy
storage
h=f(Fluid, Vel ,Distance,Temp)
Hydrodynamic boundary layer
One of the most important concepts in understanding the external flows is the
boundary layer development. For simplicity, we are going to analyze a boundary
layer flow over a flat plate with no curvature and no external pressure variation.
U U Dye streak
U U

laminar turbulent
transition
Boundary layer definition
Boundary layer thickness (d): defined as the distance away from the surface
where the local velocity reaches to 99% of the free-stream velocity, that is
u(y=d)=0.99U. Somewhat an easy to understand but arbitrary definition.
Boundary layer is usually very thin: d/x usually << 1.
Hydrodynamic and Thermal
boundary layers

As we have seen earlier,the hydrodynamic boundary layer is a region of a


fluid flow, near a solid surface, where the flow patterns are directly
influenced by viscous drag from the surface wall.
0<u<U, 0<y<d

The Thermal Boundary Layer is a region of a fluid flow, near a solid surface,
where the fluid temperatures are directly influenced by heating or cooling
from the surface wall.
0<t<T, 0<y<dt
The two boundary layers may be expected to have similar characteristics but
do not normally coincide. Liquid metals tend to conduct heat from the wall
easily and temperature changes are observed well outside the dynamic
boundary layer. Other materials tend to show velocity changes well outside
the thermal layer.
Effects of Prandtl number, Pr

d dT
dT d, dT d

Pr >>1 Pr = 1 Pr <<1
 >>  =  << 
e.g., oils e.g., air and gases e.g., liquid metals
have Pr ~ 1
(0.7 - 0.9)
u T  TW
similar to
U T  TW

(Reynold’s analogy)
Boundary layer equations (laminar flow)
• Simpler than general equations because boundary layer is thin
T
U
U
y dT
d
x TW
• Equations for 2D, laminar, steady boundary layer flow
u v
Conservation of mass :  0
x y
u u dU    u 
Conservation of x - momentum : u  v  U   
x y dx y  y 
T T   T 
Conservation of energy : u v   
x y y  y 
dU 
• Note: for a flat plate, U  is constant, hence 0
dx
Exact solutions: Blasius
d 4.99
Boundary layer thickness 
x Re x
w 0.664
Skin friction coefficient C f  1 
2 U 
2
Re x
 
 Re  U  x ,    u 
 x 
w
y 
 y 0 

 UL 
L
1 1.328
Average drag coefficient C D   C f dx   Re L  
L0 Re L   
Nu x  0.339 Re x Pr
1 1
Local Nusselt number 2 3

N u  0.678 Re L Pr
1 1
Average Nusselt number 2 3
Heat transfer coefficient
• Local heat transfer coefficient:
1 1
Nu x k 0.339k Re x Pr 2 3

hx  
x x
• Average heat transfer coefficient:
1 1
Nu k 0.678k Re L Pr 2 3

h 
L L
• Recall: qw  h ATw  T , heat flow rate from wall

• Film temperature, Tfilm


For heated or cooled surfaces, the thermophysical properties within

temperature of the wall and the free stream; T film  2 Tw  T 


the boundary layer should be selected based on the average
1
Heat transfer coefficient
Convection Thermal Boundary
Coefficient, h. Layer, dt
U

x Hydrodynamic
Boundary Layer, d

Laminar and turbulent b.l.


Laminar Region Turbulent Region
Turbulent boundary layer

* Re x increases with x. Beyond a critical value of Reynolds number


(Re x  Re xc ), the flow becomes transitional and eventually turbulent.
U  xc
Re xc  (For flow over flat plate, xc  5 105 )

* Turbulent b.l. equations are similar to laminar ones, but infinitely
more difficult to solve.
* We will mainly use correlations based on experimental data :
C f  0.059 Re x 0.2 (Re x  5 105 )

C D  0.072 Re L 
1
Re L

0.072 Re 0xc.8  1.328 Re 0xc.5 
Nu x  0.029 Re 0x.8 Pr
1
3


N u  0.036 Re 0L.8 Pr 3  Pr 3 0.036 Re 0xc.8  0.664 Re 0xc.5
1 1

Nu k
* Calculate heat transfer coefficient in usual way : h  etc.
x
Forced convection over exterior bodies
• Much more complicated.
•Some boundary layer may exist, but it is likely
to be curved and U will not be constant.
• Boundary layer may also separate from the
wall.
• Correlations based on experimental data can
be used for flow and heat transfer calculations
• Reynolds number should now be based on a  UD
characteristic diameter. Re 

D

• If body is not circular, the equivalent


diameter Dh is used 4 Area
Dh 
Perimeter
Drag force hD Nu k
CD  1 ; Nu  ; h
2  U 2
 Anormal k D
Flow over circular cylinders

Pr.62
Nu  C Re m
D
Prs.25
Re D C m
1  40 0.75 0.4
40 - 103 0.51 0.5
103 - 2  105 0.26 0.6
2  105 - 106 0.08 0.7

All properties at free stream


temperature, Prs at cylinder
surface temperature
Flow over circular cylinders

Flow patterns for cross


flow over a cylinder at
various Reynolds
numbers
FORCED CONVECTION: Internal flow
• Thermal conditions
 Laminar or turbulent
 entrance flow and fully developed thermal condition
e.g. pipe flow

Thermal entrance region, xfd,t

For laminar flows the thermal entrance length is a function of the


Reynolds number and the Prandtl number: xfd,t/D  0.05ReDPr,
where the Prandtl number is defined as Pr = / and  is the thermal
diffusitivity.
For turbulent flow, xfd,t  10D.
Thermal Conditions
• For a fully developed pipe flow, the convection
coefficient is a constant and is not varied along the
pipe length. (as long as all thermal and flow properties
are constant also.)
h(x)

constant

x
xfd,t

• Newton’s law of cooling: qS = hA(TS-Tm)


Question: since the temperature inside a pipe flow is not constant,
what temperature we should use. A mean temperature Tm is
defined.
Calculating a coefficient of convection h :
1.Calculate film temperature (if applicable)

2.Look up properties (ρ, μ, α, cp, ν, k, Pr)


3.Calculate Reynolds number Re

4.Find correct Nu correlation (BE CAREFUL!)


5.Calculate Nu using identified correlation
6.Use Nu definition to calculate convection coefficient (h)

II-80
II-81
Natural Convection
• The procedures for calculating h are very
similar with the forced convection
• calculate first the Nusselt number and from
its value the coefficient of convection
• Difference: No external source - Re cannot
be used and should be replaced by
another number: the Grashof number (Gr)

 : volume expansion coefficient


For ideal gases,

II-82
• Quite often in practice, the Grashof number and Prandtl
number appear always combined as a product.
• This is why a new number was defined, the Rayleigh
number:

II-83
II-84
Example 1
Engine oil at 60°C flows over a 5 m long flat plate
whose temperature is 20°C with a velocity of 2 m/s.
Determine the rate of heat transfer per unit width of
the entire plate.

II-85
Example 2
Air at a temperature of 30 °C and 6 kPa pressure flow
with a velocity of 10 m/s over a flat plate 0.5 m long.
Estimate the heat transferred per unit width of the
plate needed to maintain its surface at a temperature
of 45 °C.

II-86
Solution:
Properties of the air evaluated at the film temperature
o Tf = 75/2 = 37.5 oC
From air properties table at 37.5 oC:
o  = 16.95 x 10-6 m2/s.
o k = 0.027 W/m. oC
o Pr= 0.7055
o Cp= 1007.4 J/kg. oC
Note: These properties are evaluated at atmospheric
pressure, thus we must perform correction for the
kinematic viscosity
• ) act. = )atm.x (1.0135/0.06) = 2.863 x10-4 m2/s.

II-87
II-88
Example 3
Water at 25 °C is in parallel flow over an isothermal,
1-m long flat plate with velocity of 2 m/s. Calculate the
value of average heat transfer coefficient.

II-89
Example 4
A vertical 1.2 m high and 1.8 m wide double-pane window
consists of two sheets of glass separated by a 2.5 cm air gap at
atm pressure. If the glass surface temperatures across the air
gap are measured to be 20°C and 4°C, Determine the rate of
heat transfer through the window by:
a) natural convection,
b) radiation.

The effective emissivity for use in


radiation calculations between two
large parallel glass plates can be taken
to be 0.82

II-90
Mass Transfer
INTRODUCTION
Whenever there is an imbalance of a commodity in a medium, nature tends to
redistribute it until a “balance” or “equality” is established. This tendency is often referred
to as the driving force, which is the mechanism behind many naturally occurring
transport phenomena.
The commodity simply creeps away during
redistribution, and thus the flow is a diffusion
process. The rate of flow of the commodity is
proportional to the concentration gradient
dC/dx, which is the change in the concentration
C per unit length in the flow direction x, and the
area A normal to flow direction.

kdiff is the diffusion coefficient of the medium, which is a measure of how fast a
commodity diffuses in the medium, and the negative sign is to make the flow in the
positive direction a positive quantity (note2that dC/dx is a negative quantity since
concentration decreases in the flow direction).
The diffusion coefficients and thus diffusion rates of gases
depend strongly on temperature.
The diffusion rates are higher at higher temperatures.
The larger the molecular spacing, the higher the diffusion rate.
Diffusion rate: gases > liquids > solids

3
ANALOGY BETWEEN HEAT AND MASS
TRANSFER
We can develop an understanding of mass transfer in a short time with
little effort by simply drawing parallels between heat and mass transfer.

Temperature
The driving force for mass transfer is
the concentration difference.
Both heat and mass are transferred
from the more concentrated regions
to the less concentrated ones.
If there is no difference between the
concentrations of a species at
different parts of a medium, there will
be no mass transfer.

4
Conduction
Mass is transferred by conduction (called diffusion) and convection only.
Rate of mass Fick’s law of
diffusion diffusion

DAB is the diffusion coefficient (or mass


diffusivity) of the species in the mixture
CA is the concentration of the species in the
mixture

5
Heat Generation
Heat generation refers to the conversion of some form of energy such
as electrical, chemical, or nuclear energy into sensible thermal energy
in the medium.
Some mass transfer problems involve chemical reactions that occur
within the medium and result in the generation of a species throughout.
Therefore, species generation is a volumetric phenomenon, and the
rate of generation may vary from point to point in the medium.
Such reactions that occur within the medium are called homogeneous
reactions and are analogous to internal heat generation.
In contrast, some chemical reactions result in the generation of a
species at the surface as a result of chemical reactions occurring at the
surface due to contact between the medium and the surroundings.
This is a surface phenomenon, and as such it needs to be treated as a
boundary condition.
In mass transfer studies, such reactions are called heterogeneous
6
reactions and are analogous to specified surface heat flux.
Convection
Mass convection (or convective mass transfer) is the mass transfer mechanism
between a surface and a moving fluid that involves both mass diffusion and bulk
fluid motion.
Fluid motion also enhances mass transfer considerably.
In mass convection, we define a
concentration boundary layer in an
analogous manner to the thermal boundary
layer and define new dimensionless
numbers that are counterparts of the
Nusselt and Prandtl numbers.
Newton’s law
of cooling

Rate of mass
convection

hmass the mass transfer coefficient


As the surface area
Cs − C a suitable concentration difference
7
across the concentration boundary layer.
MASS DIFFUSION
Fick’s law of diffusion states that the rate of diffusion of
a chemical species at a location in a gas mixture (or
liquid or solid solution) is proportional to the
concentration gradient of that species at that location.

1 Mass Basis
On a mass basis, concentration is expressed in
terms of density (or mass concentration).

The density of a mixture at a location is equal to the


sum of the densities of its constituents at that location.

The mass fraction of a species ranges between


0 and 1, and the sum of the mass fractions8 of
the constituents of a mixture be equal to 1.
2 Mole Basis
On a mole basis, concentration is expressed in terms of molar concentration
(or molar density), which is the amount of matter in kmol per unit volume.

9
Special Case: Ideal Gas Mixtures
At low pressures, a gas or gas mixture can conveniently be approximated as
an ideal gas with negligible error.
The total pressure of a gas mixture P is equal to the sum of the partial
pressures Pi of the individual gases in the mixture..
Here Pi is called the partial pressure of species i,
which is the pressure species i would exert if it
existed alone at the mixture temperature and volume
(Dalton’s law of additive pressures).
Then using the ideal gas relation PV = NRuT where
Ru is the universal gas constant for both the species i
and the mixture, the pressure fraction of species i can
be expressed as

The pressure fraction of species i of an ideal


gas mixture is equivalent to the mole fraction
of that species and can be used in place10of it
in mass transfer analysis.
Fick’s Law of Diffusion:
Stationary Medium Consisting of Two Species
Fick’s law of diffusion

Here jdiff, A is the (diffusive)


mass flux of species A (mass
transfer by diffusion per unit
time and per unit area normal
to the direction of mass
transfer, in kg/s·m2) and
is the (diffusive) molar flux (in 11
kmol/s·m2).
Binary diffusion coefficient or mass
diffusivity, DAB: The constant of
12
proportionality in Fick’s law.
1. The diffusion coefficients, in general, are highest in gases and lowest in
solids. The diffusion coefficients of gases are several orders of magnitude
greater than those of liquids.
2. Diffusion coefficients increase with temperature. The diffusion coefficient
(and thus the mass diffusion rate) of carbon through iron during a hardening
process, for example, increases by 6000 13times as the temperature is raised
from 500°C to 1000°C.
BOUNDARY CONDITIONS
(1) specified species concentration, which corresponds to specified temperature
(2) specified species flux, which corresponds to specified heat flux.

14
STEADY MASS DIFFUSION THROUGH A WALL
Many practical mass transfer problems involve the diffusion of a species through a
plane-parallel medium that does not involve any homogeneous chemical reactions under
one-dimensional steady conditions.

diffusion resistance of 15
the wall
The rate of mass diffusion through a plane wall is
proportional to the average density, the wall area,
and the concentration difference across the wall, but
is inversely proportional to the wall thickness.

molar diffusion resistance of


the wall in s/kmol

16
Steady one-dimensional mass transfer
through nonreacting cylindrical and
spherical layers

On a molar basis

17
MASS CONVECTION
Now we consider mass convection (or convective mass transfer), which is the
transfer of mass between a surface and a moving fluid due to both mass diffusion
and bulk fluid motion.
The analogy between heat and mass convection holds for both forced and natural
convection, laminar and turbulent flow, and internal and external flow.
Mass convection is also complicated because of the complications associated with
fluid flow such as the surface geometry, flow regime, flow velocity, and the variation
of the fluid properties and composition.
Therefore, we have to rely on experimental relations to determine mass transfer.
Mass convection is usually analyzed on a mass basis rather than on a molar basis.

Concentration boundary layer: In mass


convection, the region of the fluid in
which concentration gradients exist. 18
In internal flow, we have a concentration
entrance region where the concentration
profile develops, in addition to the
hydrodynamic and thermal entry regions.
The concentration boundary layer
continues to develop in the flow direction
until its thickness reaches the tube center
and the boundary layers merge.
The distance from the tube inlet to the
location where this merging occurs is
called the concentration entry length Lc,
and the region beyond that point is called
the fully developed region.

19
A Schmidt number of near unity
(Sc = 1) indicates that momentum
and mass transfer by diffusion are
comparable, and velocity and
concentration boundary layers
almost coincide with each other.

20
The relative thicknesses of velocity, thermal, and
concentration boundary layers in laminar flow:

These relations, in general, are not applicable to turbulent boundary layers


21
since turbulent mixing in this case may dominate the diffusion processes.
22
Newton’s law
of cooling

rate of mass convection

for internal flow

average mass
transfer coefficient

23
hmass is the mass transfer coefficient
DAB is the mass diffusivity.
The Nusselt and Sherwood numbers
represent the effectiveness of heat
and mass convection at the surface,
respectively.

The Sherwood number can be obtained


from the Nusselt number expression by
simply replacing the Prandtl number by
the Schmidt number.
For natural
convection24
mass transfer
25
Natural
convection
mass transfer

Applicable to both temperature- and/or


concentration-driven natural convection flows.

26
Analogy between Friction, Heat Transfer, and Mass
Transfer Coefficients

27
Special Case: Pr  Sc  1 (Reynolds Analogy)

This relation is known as the Reynolds


analogy, and it enables us to determine the
seemingly unrelated friction, heat transfer,
and mass transfer coefficients when only
one of them is known or measured.

28
To develop the equations which describe pollutant concentration in the plug of fluid
as it flows down the PFR,

Conduct a mass balance on a control volume which encloses a section of the PFR of
infinitesimally small thickness dx, as shown.

Since the thickness is small, we can assume that the fluid in that region of the PFR is
well-mixed. The mass balance equation for this control volume is We have set equal
to zero, indicating that this is a steady-state problem. We are assuming here that
Plug-Flow Chemical Reactor

Qi, CAi Qo, CAo


CA(z) Dz

z A

k
B
r  kCA
• Assumptions
o Pure reactant A in feed stream
o Perfect plug flow
o Steady-state operation
o Isothermal operation
o Constant physical properties (r, k)
Plug-Flow Chemical Reactor cont.

Qi, CAi Qo, CAo


CA(z) Dz

z
• Overall mass  Component balance
balance (QC A ) z  (QC A ) z  Dz  kC A ADz  0
     
( r Q ) z  ( r Q ) z  Dz  0
    A in A out A consumed
 Q ( C A ) z  ( C A ) z  Dz 
Mass in Mass out
lim   kC A   0
 (Q ) z  (Q ) z  Dz  
Dz  0 A Dz 
lim   0
Dz  0
 Dz  
Q dC A
 kC A  0
dQ A dz
 0 Q dC A
dz  kC A  0 C A (0)  C Ai
Qi  Q0  Q A dz
Practice problems

1. Consider an accidental release of benzene (C6H6, a carcinogenic substance) in a river


moving at the uniform speed of 15 cm/s. From water, benzene volatilizes into the air,
at a rate that corresponds to a decay constant of 0.20/day (a function of water depth).
If the initial concentration reaches a maximum of 0.025 mg/L at the center of the spill,
and the drinking standard is 0.005 mg/L, which downstream length of the river will be
subjected at some time or other to a concentration exceeding the drinking standard
and how long will the episode last? Assume first-order chemical kinetics.

2. Show that, for steady incompressible


laminar flow between two parallel plates
depicted in Figure below, the conservation
of momentum reduces to

The flow is assumed to be fully developed


and steady and external forces are
negligible.
Solve the equation for u using the following boundary conditions: ,

3. Water at 20 °C flows between two large stationary parallel plates which are 2 cm apart.
If the maximum velocity is 1m/s, determine (a) average velocity, (b) the velocity
gradients at the plates and (c) the difference in pressure between two points 10 m
apart. Viscosity of water at 20 °C is 0.001 Pa-s. Consider the flow to be a fully developed
one.

4. A mixture of He and N2 gas is contained in a pipe at 298 K and 1 atm total pressure
which is constant throughout. At one end of the pipe at point (1) the partial pressure
pA1 of He is 0.60 atm and at the other end 0.2 m, pA2 = 0.20 atm. Calculate the flux of
He at steady state if DAB of the He-N2 mixture is 0.687 cm2/s.
5. Heat conduction in an annulus.
(a) Heat is flowing through an annular wall of inside
radius r0 and outside radius r1 as in the Figure. The
thermal conductivity varies linearly with temperature
from k0 at T0 to k1 at T1. Develop an expression for the
heat flow through the wall.
(b) Show how the expression in (a) can be simplified
when r1 − r0 is very small. Interpret the result physically.

6. Review the application of shell balance theory for flow through an annulus found in the
text book (BSL, page 57).
Practice problems

1. Consider an accidental release of benzene (C6H6, a carcinogenic substance) in a river


moving at the uniform speed of 15 cm/s. From water, benzene volatilizes into the air,
at a rate that corresponds to a decay constant of 0.20/day (a function of water depth).
If the initial concentration reaches a maximum of 0.025 mg/L at the center of the spill,
and the drinking standard is 0.005 mg/L, which downstream length of the river will be
subjected at some time or other to a concentration exceeding the drinking standard
and how long will the episode last? Assume first-order chemical kinetics.

2. Show that, for steady incompressible


laminar flow between two parallel plates
depicted in Figure below, the conservation
of momentum reduces to

The flow is assumed to be fully developed


and steady and external forces are
negligible.
Solve the equation for u using the following boundary conditions: ,

3. Water at 20 C flows between two large stationary parallel plates which are 2 cm apart.
If the maximum velocity is 1m/s, determine (a) average velocity, (b) the velocity
gradients at the plates and (c) the difference in pressure between two points 10 m
apart. Viscosity of water at 20 C is 0.001 Pa-s. Consider the flow to be a fully developed
one.

4. A mixture of He and N2 gas is contained in a pipe at 298 K and 1 atm total pressure
which is constant throughout. At one end of the pipe at point (1) the partial pressure
pA1 of He is 0.60 atm and at the other end 0.2 m, pA2 = 0.20 atm. Calculate the flux of
He at steady state if DAB of the He-N2 mixture is 0.687 cm2/s.
5. Heat conduction in an annulus.
(a) Heat is flowing through an annular wall of inside
radius r0 and outside radius r1 as in the Figure. The
thermal conductivity varies linearly with temperature
from k0 at T0 to k1 at T1. Develop an expression for the
heat flow through the wall.
(b) Show how the expression in (a) can be simplified
when r1 − r0 is very small. Interpret the result physically.

6. Review the application of shell balance theory for flow through an annulus found in the
text book (BSL, page 57).
1. The General Continuity Equation can be written in terms of concentration c as
follows:

(1)

where s is the source of the quantity c per unit volume and unit time.

In such case, the ‘source’ term s is properly speaking a ‘sink’ and has a negative value.
Under first-order chemical kinetics, the rate of removal of the chemical is proportional
to its own concentration.

s = −Kc, where K is a constant of decay, with dimension equal to the inverse of time.

Considering a one-dimensional system with uniform and constant velocity u, the


continuity Equation becomes:

(2)

To solve this equation, we need to know the initial concentration of the pollutant.
Let us denote it as c(x, t = 0) = c0(x).

If there were no chemical degradation (K = 0), this patch would simply move
downstream without change, and the solution would be c(x, t) = c0(x − ut), which
corresponds to a mere translation down the x–axis by the traveled distance ut. On the
other hand, if there were no movement (u = 0) and only chemical degradation, the
solution would be c(x, t) = exp(−Kt) c0(x), which is the initial distribution attenuated
over time. Combining these two limiting solutions, it is not difficult to show that the
distribution of concentration corresponding to the combined case of advection and
degradation at any later time t is given by

(3)

 Now, we track the maximum of the concentration over time. Equation (3) tells
that the maximum at time t is the initial maximum times the factor exp(−Kt).
 The contamination episode ends when the maximum concentration falls to the
drinking standard and thus at time t such that (0.005 mg/L) = exp(−Kt)× (0.025
mg/L).
 The solution is exp(−Kt) = 0.005/.025 = 0.20, giving Kt = 1.609 and t = 8.047
days =6.953 × 105 s.
 Over this time, the river water has traveled a distance ut = (0.15 m/s)(6.953 ×
105 s)= 104.3 km. Thus, more than 100 kilometers of river are being
contaminated, and the episode will last slightly more than 8 days.
2.
3.
4.
5.

Вам также может понравиться